You are on page 1of 53

Oculoplastics/Orbit

Question 1 of 130
Indications for closure of carotid cavernous sinus fistulas does not include which of the following?

Ophthalmoplegia

Severe headache

Persistent bruit

Progressive proptosis
Please select an answer
Feedback: The usual treatment of carotid cavernous sinus fistulas is through interventional radiology. The primary
technique for closure of these fistulas is placement of thrombogenic agents into the arterial system. A recently developed
technique for gaining access to the shunts involves cannulating the superior ophthalmic vein. Cavernous sinus and dural
sinus fistulas can produce several signs that are visually or otherwise functionally debilitating to the individual. Severe
proptosis potentially subjects the patient to corneal injury. Ophthalmoplegia from neural ischemia or vascular engorgement
of the orbit is a severe sequela of these fistulas. Severe headaches may be functionally debilitating to the patient. Bruits
mayor may not be noted by the patient and would not in isolation be considered an indication for ablation of the fistula.

Question 2 of 130
Which of the following findings is most commonly associated with orbital floor fractures?

Orbital cellulitis

Enophthalmos within 24 hours of the fracture

Rapid improvement in traumatic diplopia over a 24-hour period

Unilateral midfacial sensory loss (V2 distribution unilaterally)


Please select an answer
Feedback: A common site of orbital floor fractures is the thin bony encasement of the infraorbital nerve in the orbital floor.
Infraorbital nerve trauma leads to decreased sensation in the midface and is observed frequently. Orbital cellulitis is a rare
occurrence in this setting. Enophthalmos may develop in larger fractures as orbital edema decreases; however, it usually
takes longer than 24 hours for this to become evident. Similarly, the diplopia occurring after a floor fracture may result from
entrapment or contusion of the muscle. In either case, complete resolution would be rare within 24 hours.
Question 3 of 130
What is the most common complication of external lower eyelid blepharoplasty?

Lower eyelid retraction

Pyogenic granuloma at the incision site

Lash loss

Bacterial infection at the incision site


Please select an answer
Feedback: The most common complication of lower eyelid blepharoplasty is lower eyelid retraction. Retraction occurs from
inadequate assessment of lateral canthal laxity during the preoperative examination, aggressive removal of lower eyelid
skin, or postoperative middle eyelid lamella scarring. Careful attention should be drawn to lateral canthal tendon laxity during
the initial examination. If laxity exists, it should be corrected surgically during the blepharoplasty. Overzealous removal of
skin can cause retraction and is difficult to repair. Some reduction in scar formation may be seen from injection of steroids
into the lower eyelids. Frequently, repair will require skin grafting to the lower eyelid, which nullifies any cosmetic improve-
ment initially desired by the patient. Some postoperative scarring may be unavoidable and may overcome the lateral and
medial canthal abilities to maintain the normal lower eyelid position. Pyogenic granuloma may occur, particularly if a
transconjunctival approach is used. However, these granulomas are neither common nor serious complications. Lash loss
should not occur, as surgical incisions are relatively far from the lash follicles. Bacterial infection after blepharoplasty is rare
unless the patient is immunocompromised (eg, the patient has insulin-dependent diabetes).

Question 4 of 130
What is the most appropriate treatment of a non-painful, well-circumscribed homogenous spherical mass in the inferior orbit
that moderately enhances with contrast in an otherwise healthy patient who complains of diplopia?

Chemotherapy

Radiation therapy

Ssurgical excision

Observation
Please select an answer
Feedback: Cavernous hemangiomas are benign lesions. Many lesions are diagnosed on orbital CT scans ordered for other
reasons, such as headaches. Symptomatic lesions should be removed by surgical excision. Such lesions can be managed
conservatively with serial examinations and observation if signs and symptoms such as proptosis, visual loss, and diplopia
do not exist. Chemotherapy and radiotherapy are not necessary for these benign lesions.
Question 5 of 130
Which of the following statements does not accurately describe a sebaceous cell adenocarcinoma lesion?

The lower eyelid is more frequently involved than the upper eyelid.

Radiation therapy is thought to be a causative factor.

Most arise from the meibomian glands of the tarsus.

The initial course is indolent and often misdiagnosed.


Please select an answer
Feedback: Sebaceous cell adenocarcinoma is a rare adnexal tumor with a predilection for eyelid skin. It represents 1%-3%
of all malignant eyelid lesions. Sebaceous cell carcinoma arises from the meibomian glands of the tarsus, which are more
numerous on the upper than the lower eyelid. They may also arise from the glands of Zeis as well as the sebaceous glands
in the caruncle and eyebrows. The upper eyelid is affected twice as commonly as the lower eyelid. In 10% of cases, both
upper and lower eyelid involvement is present. Radiotherapy of the head and neck, as well as chronic chalazia, are thought
to be causative factors. Clinical presentation may include a chronic chalazion, blepharitis, blepharoconjunctivitis, or a firm,
painless nodule on the inner surface of the eyelid. The presence of unilateral blepharitis and recurrent chalazia in the same
location is an indication for further investigation and perhaps eyelid biopsy. The clinical differential diagnosis includes
chalazia, blepharitis, basal and squamous cell carcinoma, pyogenic granuloma, and ocular cicatricial pemphigoid.

Question 6 of 130
Which of the following sequelae of a carotid cavernous sinus fistula is the most common cause of visual disability?

Strabismus

Proptosis with corneal exposure

Spontaneous choroidal detachment

Elevated intraocular pressure with progressive optic nerve damage


Please select an answer
Feedback: Visually disabling effects of a carotid cavernous sinus fistula include secondary glaucoma, spontaneous
choroidal detachment, strabismus, proptosis with corneal exposure, and retinal and optic nerve ischemia. Secondary
glaucoma is the most common cause of visual loss. Unfortunately, this form of secondary glaucoma does not respond well
to the traditional medical and surgical therapy and is an indication for neuroradiologic intervention.
Question 7 of 130
A patient undergoes an uncomplicated enucleation for a blind, painful eye. A hydroxyapatite implant is placed in the socket.
Six weeks after surgery, the patient's examination shows a well-healed socket with a deep superior and inferior fornix.
Movement of the orbital implant is excellent. However, the patient is disappointed that the prosthesis does not move well
and asks if any improvements are possible. You discuss the option of a plastic peg in the implant, which can be coupled to
the prosthesis and improve prosthesis movement. What is the most appropriate next step?

Schedule the patient for the next available surgical date

Order MRI with contrast of the orbit

Order a bone scan of the orbit

Schedule the patient for a return visit in 3 months


Please select an answer
Feedback: Placement of a plastic peg into an hydroxyapatite implant can significantly improve the movement of the
prosthesis. Drilling a hole into the center of the implant is a short procedure. Vascular ingrowth into the implant allows
epithelial tissue to line the peg hole and provides stabilization for the peg. Adequate vascular ingrowth usually occurs 4 to 6
months after the implant is placed in the socket. Confirmation of this vascularization can be obtained with a bone scan or
MRI with contrast. If the peg is placed prior to vascularization, there is an increased risk of peg destabilization and bacterial
infection in the hydroxyapatite implant. A prudent recommendation would be to see the patient in 3 months and order one of
the suggested scans at that time. If vascularization is present, then proceeding with peg placement would be an appropriate
course to improve prosthesis movement.

Question 8 of 130

Which of the following is not a component of Horner's syndrome?

Miosis

Anhidrosis

Blepharoptosis

Decreased stimulation of the levator muscle


Please select an answer
Feedback: Miosis is a common finding in Horner's syndrome. This diagnosis can be confirmed by failure of the pupil to
dilate after instillation of topical 10% cocaine drops. Cocaine acts by blocking reuptake of norepinephrine and in Horner's
syndrome, poor dilation occurs because little norepinephrine is being released into the synaptic cleft. After the effects of the
cocaine are eliminated (several days), localization of the Horner's lesion can be refined with topical 1%
hydroxyamphetamine (Paredrine). Paredrine acts by releasing norepinephrine from the presynaptic terminal, and in a pre-
ganglionic Horner's lesion, dilation will occur. Dilation will not occur in a postganglionic lesion because the nerve terminal will
have degenerated. Anhidrosis is a less common finding and suggests a lesion below the carotid artery bifurcation in the
neck. Horner's syndrome affects the sympathetic stimulation of Muller's muscle, resulting in decreased stimulation and a
lowered position of the upper eyelid. The levator muscle is innervated through cranial nerve III. The loss of stimulation to
Muller's muscle provides the basis for surgical correction of blepharoptosis. Elevation of the eyelid can be easily achieved by
performing a conjunctival Muller's muscle resection.
Question 9 of 130
A patient is evaluated preoperatively for blepharoplasty surgery. Which of the following is not a relative contraindication to
surgery?

Poorly controlled hypertension

Severe keratoconjunctivitis sicca

Insulin-dependent diabetes

Atrial fibrillation with anticoagulation


Please select an answer
Feedback: Patients undergoing blepharoplasty surgery with moderately dry eyes may develop debilitating exposure after
this surgery. This result may be due to marginally compensated dry eyes, which become problematic due to poor
postoperative lid closure or lagophthalmos. Anticoagulation is a strong contraindication to blepharoplasty, particularly if
orbital fat is to be removed. Postoperative retro-orbital bleeding may produce compression of the optic nerve and visual loss.
This may occur also in cases of elevated intraoperative and postoperative blood pressure. Although insulin-dependent
diabetes may delay wound healing and predispose patients to infection, it is not a relative contraindication to surgery.

Question 10 of 130
A 25-year-old man is seen in your office 2 months after blunt trauma to the right orbit. The examination is normal except for
blepharoptosis on the right side. Levator function is normal on both sides, and the patient states the eyelid positions were
equal on both sides prior to the injury. There is no enophthalmos, and the patient does not complain of diplopia. What is the
best next step in managing this patient?

Surgical exploration and repair of the levator aponeurosis

Close observation with no plan for surgical correction until 3 to 6 months after initial injury

Computed tomography (CT) scan to rule out an orbital fracture

A Tensilon test to rule out new-onset myasthenia gravis


Please select an answer
Feedback: Blunt trauma to the orbit commonly results in edema and ecchymosis to the periorbital soft tissue. These latter
processes may result in stretching, dehiscence, or disinsertion of the levator aponeurosis. It is difficult to correlate the
degree of injury to the levator aponeurosis with the severity of blunt injury. Younger patients tend to have enough elasticity in
the tissue to allow significant if not complete resolution of the blepharoptosis. However, for all patients, blepharoptosis from
blunt injury should be allowed the opportunity to spontaneously resolve without early surgical intervention. Patients should
be examined monthly for changes in the eyelid position. If no improvement in eyelid position occurs by 6 months after the
injury, it is unlikely that any improvement will occur in the future. In this case, surgical correction may be planned. However,
if improvement is noted on a month-to-month basis, then surgery should be delayed until improvement reaches a plateau or
all blepharoptosis resolves. A CT scan is helpful in evaluating injury to adjacent bones. There is no evidence of injury to
orbital bones based on the exam (ie, no diplopia or enophthalmos). Given the history of blunt trauma, it is unlikely that a
Tensilon test is warranted. Myasthenia gravis should be considered in the diagnosis of adult blepharoptosis in the absence
of other predisposing factors.
Question 11 of 130
A 61-year-old man presents with a 1-week history of redness and pain of the right eye. He wonders if this is related to his
chronic sinus problems. On examination, his visual acuity is 20/20. His right upper eyelid is swollen. The right conjunctiva is
injected with dilated episcleral vessels inferiorly. The underlying sclera appears inflamed. His ocular motility is limited, and
there is 2 mm of proptosis OD. A CT scan shows a diffuse infiltrate in the right inferior orbit. There is also thickening of the
left nasal mucosa. Which of the following tests would be most beneficial in diagnosing this patient's condition?

Serum c-ANCA (antineutrophil cytoplasmic antibodies)

Serum p-ANCA (antineutrophil cytoplasmic antibodies)

Serum ESR (erythrocyte sedimentation rate) and C-reactive protein

Conjunctival culture for bacterial and viral pathogens


Please select an answer
Feedback: In a patient who presents with scleritis or episcleritis, orbital involvement, and a history of sinus or nasal disease,
the diagnosis of Wegener's granulomatosis must be considered. Eye involvement is found in up to 50% of patients with
Wegener's granulomatosis. Orbital findings are most common and appear in approximately 22% of patients. Typical orbital
findings include proptosis, lid swelling, ophthalmoplegia, and pain. CT scanning shows a diffuse, often multicentric orbital
infiltrate. Associated sinus and nasal involvement is evident in 50% of cases with orbital disease. Bony destruction of the
sinuses is typical. Scleritis and episcleritis may be seen in approximately 20% of patients. In 10% of patients, ocular
complaints are the initial manifestation of the disease. In this case, a serum c-ANCA would be most helpful, considering the
patient's presentation. The c-ANCA has been shown to be a sensitive and specific test for the diagnosis of Wegener's
granulomatosis. The p-ANCA titer is not specific for Wegener's granulomatosis and need not be ordered. Sarcoidosis should
be considered in the differential, but diffuse orbital involvement is uncommon. The lacrimal gland is the most common orbital
structure affected in sarcoidosis. An ACE level is nonspecific but is helpful in the diagnosis and treatment of sarcoidosis. An
ESR and C-reactive protein may be elevated in this case but are nonspecific. Conjunctival cultures are not indicated.

Question 12 of 130
A 52-year-old nondiabetic woman presented with a 4-day history of right eyelid swelling, conjunctival injection, and pain. She
stated that she had awoken that morning with double vision. On examination, her visual acuity was 20/20 with correction.
There was no relative afferent pupillary degect. There was a small abduction deficit OD. The right upper eyelid was
erythematous and swollen. The conjunctiva was injected and chemotic. Deeper episcleral vessels were also injected. The
cornea was clear and the anterior chamber was deep and quiet. The external examination of the left eye was
normal. Funduscopic examination was unremarkable OU. There was 2 mm of proptosis OD. You ordered a CT scan of the
orbits, which showed diffuse soft tissue infiltrate involving the anterior portion of the right orbit. The nasal mucosa appeared
thickened on the left side. The patient was afebrile, and laboratory studies, including CBC with differential, were all normal.
After 7 days on oral corticosteroids, the patient shows moderate improvement. She has less pain, and redness and eyelid
swelling have decreased. However, the proptosis and diplopia are unchanged. Which of the following is the most
appropriate next step in the management of this patient's condition?

Start intravenous corticosteroids

Continue the present treatment for 1 more week

Remove tissue from the right orbit for biopsy

Repeat the CT scan of the orbits


Please select an answer
Feedback: Any orbital inflammatory pseudotumor that responds poorly or incompletely to adequate steroids should prompt
the ophthalmologist to consider other diagnoses. In this case, an orbital biopsy is indicated because of a lack of dramatic
improvement in proptosis and diplopia after 1 week of steroids. A systemic workup may also be required, depending on the
biopsy results. Vasculitic, neoplastic, and infectious processes must be considered. The systemic vasculitides, including
Wegener's granulomatosis polyarteritis nodosa, giant cell arteritis, and hypersensitivity vasculitis (associated with systemic
lupus erythematosus, rheumatoid arthritis, drug reactions, malignancies, and chronic infection) may all cause orbital
inflammation identical to inflammatory pseudotumor. Orbital lymphoma may also respond transiently to corticosteroids with
reduction in size, but it will continue to grow over time. To rule out infectious causes, biopsy specimens should be examined
with special stains for acid-fast bacillus, fungus, and treponema1 pathogens. Fresh tissue should be sent for lymphocytic
markers. Orbital or periorbital abscesses are unlikely to develop in the presence of such clinical improvement as is seen in
this patient. Repeating orbital CT scans is unlikely to provide new information because proptosis is not reduced. Changing
the route of steroid administration or continuing its use after 1 week with no reduction in proptosis may delay diagnosis.

Question 13 of 130
Which of the following would not cause discharge in patients with an anophthalmic socket and ocular prosthesis?

An old prosthesis

Dry socket

Bacterial conjunctivitis

Eyelid malposition
Please select an answer
Feedback: Patients with ocular prostheses frequently complain of chronic production of a mucoid discharge emanating from
the eyelids. The most common cause of discharge is a dry eye, which is treated with lubricating drops and ointment.
Bacterial or viral conjunctivitis may cause discharge, but in anophthalmic patients this cause is rare. As ocular prostheses
age, the synthetic components degrade, causing microscopic irregularities on the surface of the prosthesis. These surface
changes may be abrasive to the upper eyelid as it passes over the prosthesis and may lead to conjunctival irritation and
mucoid discharge. Ectropion resulting from gravitational traction on the canthal tendons due to the weight of the prosthesis
can occur and may result in exposure of the lower eyelid palpebral conjunctiva, but this does not produce discharge

Question 14 of 130
If the base curve of a soft contact lens is held constant, but the lens diameter is increased, what would one expect to see?

Circumferential limbal injection

Excessive lens movement

A looser fit

Contact edge lift


Please select an answer
Feedback: Holding the base curve constant and increasing the lens diameter will tighten the fit of the contact lens on the
eye. With a tighter fit, one might see compromise of the limbal area with increased injection. In contrast, signs of a loose fit
include edge lift, excessive lens movement, and decentration of the lens.
Question 15 of 130
For the severely and profoundly visually impaired, which of the following is most correlated with successful employment and
occupational rehabilitation?

Skill with use of a white cane

Braille literacy

Higher education

Personal grooming skills


Please select an answer
Feedback: Severely and profoundly visually impaired individuals (such as those who are legally blind) may have
experienced sight loss in childhood or during their adult employment years. These people will need professional and/or
vocational training to (re)establish skills and acquire adaptive methodologies to meet the demands of their jobs. Each state
provides rehabilitative services to the visually impaired, usually via the state's Commission for the Blind (or a similarly titled
organization), and this service often begins with the ophthalmologist's referral of the adult patient to this commission. Each
of the skills listed is highly valued, and physicians may debate the preferred response; however, within the community of
successfully employed blind and visually impaired, where the underemployment rate approaches 75%, Braille skills rank as
the most valuable asset. Orientation and mobility skills are extremely valuable in meeting employer expectations and
personal travel demands. Naturally, the visually impaired must also maintain appropriate and acceptable appearance
standards. Regarding the correlation of successful employment with higher education, a highly trained individual will suffer
severe performance regression with the onset of the visual impairment unless access to printed material and an efficient
method of expressive creativity is reestablished. Computers with specially designed word processors, optical text scanners,
and speech synthesizers are useful, but are generally insufficient to meet all the demands of these types of employees. For
individuals who enter vocational or professional school with a preexisting visual impairment, successful program completion
and subsequent employment is most associated with Braille literacy. Realistically, employers will pay the visually impaired a
full-day's wage only if they can perform a full-day's work and compete with their sighted peers. In addition, as has been
revealed in focus groups with blind workers, the expectations placed on these people by their loved ones, their employers,
and their vocational rehabilitation counselors are very important in helping them reach their greatest occupational potential.

Question 16 of 130
The use of guide dogs or Seeing Eye dogs by the blind is a personal choice. Which of the following statements most
accurately describes guide dogs?

Guide dogs are typically matched with individuals for the life of the animal, often working for more than 10 years.

Guide dogs are most commonly used by legally blind people, not exclusively by totally blind individuals.

Typically, individuals obtain orientation and mobility training for personal travel either on the long white cane or with a
guide dog, one or the other.

There are specific restrictions regarding guide dogs that limit their access to special public areas, such as food
preparation establishments and secured areas of airports.
Please select an answer
Feedback: Guide dogs are closely matched with individuals after an extensive indoctrination program for the blind candidate
and a supervised 2-year adolescence followed by a rigorous training program for the canine. The average guide dog works
for 6 to 10 years, followed by retirement. Fifty percent of guide dog owners retain their retired guide as a family pet; the other
half give their guides for adoption. Nearly all individuals replace the retiring dog with a new and properly matched younger
animal. While accompanying their owners, guide dogs are legally permitted to enter any and all public facilities without
restriction. As a matter of guide dog courtesy, however, many blind users elect not to use guides at zoos for obvious
reasons related to mutual and unfavorable animal distractions. Because only 10% of the visually impaired are near-totally or
totally blind, it is not surprising to learn that the majority of guide dog users (80%) have measurable visual acuities. Any
guide dog user must acquire excellent white cane travel skills before obtaining the guide dog because these animals are
occasionally not available for work because of visits to the veterinarian.

Question 17 of 130
Which of the following is not a reported complication of hydroxyapatite implants?

HIV transmission from the donor sclera

Exposure of the hydroxyapatite surface

Chronic bacterial infection in the implant

Migration and extrusion of the implant


Please select an answer
Feedback: The primary complications of placement of a hydroxyapatite implant in an anophthalmic orbit are exposure and
extrusion. Rates of exposure are as high as 10%-12%. Exposure is often treated by observation, allowing granulation and
closure of the defect, or by surgical closure with undermining, advancement, or grafting of tendons and conjunctival tissue.
Chronic bacterial infection has occurred and is difficult to treat. In some cases it requires complete removal of the implant.
HIV transmission from sclera used to wrap the implant remains a concern for many patients. At this time, however, there
have been no reported cases of HIV transmission to a patient from sclera used in this procedure. Patient concerns may
require using an alternate material (eg, polygalactin mesh) to wrap the implant or using a separate implant system.

Question 18 of 130
A 52-year-old nondiabetic woman presents with a 4-day history of right eyelid swelling, conjunctival injection, and pain. She
states that she awoke this morning with double vision. On examination, her visual acuity is 20/20 with correction. There is no
relative afferent pupillary defect. There is a small abduction deficit OD. The right upper eyelid is erythematous and swollen.
The conjunctiva is injected and chemotic. Deeper episcleral vessels are also injected. The cornea is clear and the anterior
chamber is deep and quiet. The external examination of the left eye is normal. Funduscopic examination is unremarkable
OU. There is 2 mm of proptosis OD. You order a CT scan of the orbits, which shows a diffuse soft tissue infiltrate involving
the anterior portion of the right orbit. The nasal mucosa appears thickened on the left side. The patient is afebrile, and
laboratory studies, including CBC with differential, are all normal. Which of the following actions is not appropriate in the
management of this patient's condition?

Begin a topical antibiotic/corticosteroid combination

Perform an orbital biopsy

Begin oral prednisone 60 mg/day with follow-up in the morning

Begin oral antibiotics with follow-up in 3 days


Please select an answer
Feedback: This patient has a fairly typical idiopathic orbital inflammation (orbital pseudotumor). Although some physicians
would elect to start the patient on a trial of corticosteroids, others would prefer to have a tissue diagnosis prior to the
initiation of therapy. Because the patient has no fever or elevated white blood cell count, the diagnosis of an infectious
orbital cellulitis is unlikely. However, a course of antibiotics in a patient with fairly mild orbital symptoms would not be
unreasonable. The use of topical medication in this case would have little effect.
Question 19 of 130
During a consultation in the emergency room, you are asked to see a patient with noninfected second-degree burns to the
eyelids. The emergency room physicians ask what topical medicine should be placed over the burns to prevent scarring.
What would the most appropriate response be?

Gentamicin in a water-miscible base

Silver sulfadiazine 1% cream

No topical medications

Hydrocortisone 1% ointment
Please select an answer
Feedback: During the acute phases of a first-or second-degree burn, topical medications are not indicated. If full-thickness
or third-degree burns are present, the tissue should be covered with silver sulfadiazine 1% in a water-miscible base or a
nonsensitizing antibiotic ointment such as gentamicin in a water-miscible base. Cultures should be taken at the end of the
acute phase to diagnose the presence of bacterial contamination. Infected second-degree burns should also be treated with
the water-miscible medicines just mentioned. Cool, saline-soaked gauze pads may be beneficial in the acute stages of a
second-degree burn. There is no role for topical steroids in eyelid burns.

Question 20 of 130

A 44-year-old patient's spectacle correction is -8.00 sphere with a vertex distance of 16 mm. Which contact lens power
should be ordered for this patient?

-8.00

-7.00

-7.50

-8.50
Please select an answer
Feedback: If the spherical correction for a patient is greater than 4 D, the vertex distance needs to be taken into
consideration because of lens effectivity. A lens (whether plus or minus) loses plus power (or gains minus power) as it is
moved toward the cornea. The proper power can be determined from a conversion table, or mathematically: 1/8.00 D =
0.125m = 125mm; 125mm +16 mm = 141mm = 0.141m; 1/0.141m = 7.1 D. A -7.00 sphere lens is closest in power. This
patient should not be "overminused" with a -7.50 contact lens to provide better distance vision. In a 44-year-old patient,
switching from glasses to contact lenses with a highminus prescription is likely to exacerbate presbyopic symptoms, and
overminusing this patient will only make him more symptomatic.
Question 21 of 130
A 20-year-old man is seen in the emergency room after right medial canthal injury from a sharp object. Examination reveals
a 2-cm vertical laceration over the medial canthus. The patient appears to have a complete avulsion of the medial canthal
tendon and mild telecanthus. No evidence of fracture is seen on a CT scan of the orbits. The patient is taken to the
operating room so the medial canthal tendon can be reattached. What is the most important point of attachment to recreate
a normal eyelid appearance?

The posterior lacrimal crest

The fundus of the lacrimal sac

The center of the lacrimal fossa

The anterior lacrimal crest


Please select an answer
Feedback: A clear understanding of medial canthal anatomy will help resolve this problem. The medial canthal tendon
converges from upper and lower limbs to divide into anterior and posterior arms, which attach to the respective lacrimal
crests. The anterior lacrimal crest attachment is diffuse and strong. The posterior lacrimal crest attachment is more delicate
but very important in maintaining proper curvature of the eyelids at the medial canthus. Reattachment of the tendon to the
anterior crest alone will distort the eyelid position by creating an anteriorly displaced flat appearance rather than a natural
convex appearance for the eyelid. Attachment to the posterior crest creates the most natural appearance. The lacrimal sac
fundus is situated in the lacrimal fossa between the anterior and posterior arms of the tendon. Placement of sutures in this
area increases the risk of injury to the lacrimal sac, resulting in nasolacrimal obstruction.

Question 21 of 130
A 20-year-old man is seen in the emergency room after right medial canthal injury from a sharp object. Examination reveals
a 2-cm vertical laceration over the medial canthus. The patient appears to have a complete avulsion of the medial canthal
tendon and mild telecanthus. No evidence of fracture is seen on a CT scan of the orbits. The patient is taken to the
operating room so the medial canthal tendon can be reattached. What is the most important point of attachment to recreate
a normal eyelid appearance?

The posterior lacrimal crest

The fundus of the lacrimal sac

The center of the lacrimal fossa

The anterior lacrimal crest


Please select an answer
Feedback: A clear understanding of medial canthal anatomy will help resolve this problem. The medial canthal tendon
converges from upper and lower limbs to divide into anterior and posterior arms, which attach to the respective lacrimal
crests. The anterior lacrimal crest attachment is diffuse and strong. The posterior lacrimal crest attachment is more delicate
but very important in maintaining proper curvature of the eyelids at the medial canthus. Reattachment of the tendon to the
anterior crest alone will distort the eyelid position by creating an anteriorly displaced flat appearance rather than a natural
convex appearance for the eyelid. Attachment to the posterior crest creates the most natural appearance. The lacrimal sac
fundus is situated in the lacrimal fossa between the anterior and posterior arms of the tendon. Placement of sutures in this
area increases the risk of injury to the lacrimal sac, resulting in nasolacrimal obstruction.
Question 22 of 130
A 30-year-old man is evaluated in the emergency room for trauma to the right orbit. The patient has marked proptosis and
an intraocular pressure of 40 mm Hg on the affected side. A CT scan shows intraorbital hemorrhage. Which of the following
actions would be the least effective in acutely reducing intraocular pressure?

Lateral canthotomy and cantholysis

Administration of high-dose oral corticosteroids

Administration of topical aqueous suppressants

Administration of intravenous mannitol


Please select an answer
Feedback: Orbital blood vessels are suspended within the orbital fat by fibrous septa. Thus, orbital fat cushions many
vessels in the anterior and central part of the orbit. Vessels exiting the orbital foramen, such as ethmoidal arteries and veins
or vessels entering the optic canal and orbital fissures, are less protected and are subject to shearing forces during injury.
Acute onset orbital hemorrhage can produce a sudden elevation of intraorbital and intraocular pressure. Rapid treatment of
this pressure reduces the risk of central retinal artery occlusion. Early intervention includes administration of topical aqueous
suppressants, oral or intravenous acetazolamide, and intravenous mannitol. Performing a lateral canthotomy and
cantholysis reduces pressure by allowing the globe to move forward without the restraint of eyelid tissue. Ultimately, the
surgeon is faced with the dilemma of evacuating the hemorrhage versus waiting for spontaneous resolution. During the
acute phase, intravenous corticosteroids may be more helpful than oral corticosteroids because of the more rapid onset of
action.

Question 23 of 130
A 2-year-old girl is referred to your office for evaluation of left lower eyelid ecchymosis. There is 3 mm of proptosis of the left
eye. Her medical history is significant for treatment of some unknown tumor. Which of the following childhood tumors is the
most likely diagnosis?

Rhabdomyosarcoma

Retinoblastoma

Neuroblastoma

Leukemia
Please select an answer
Feedback: Proptosis in a child indicates an orbital process that may have infectious, inflammatory, and neoplastic causes.
This patient has only eyelid ecchymosis and no signs of orbital inflammation. Retinoblastoma presents as an intraocular
tumor with the potential for orbital spread. Even though this tumor can present with orbital involvement, the condition occurs
usually in underdeveloped countries and is diagnosed in later stages. Neuroblastoma often presents with proptosis, globe
displacement, and eyelid ecchymosis. In the majority of cases, these orbital tumors represent metastasis from the primary
abdominal tumor. These patients have increased urine levels of vanillylmandelic acid (VMA). Treatment of this metastatic
orbital disease is a combination of radiation and chemotherapy. Leukemia can lead to orbital involvement with proptosis.
The most frequent form of leukemia to affect the orbit is acute myelogenous leukemia (chloroma). Eyelid ecchymosis is not
a common finding in orbital involvement with leukemia or rhabdomyosarcoma. Rhabdomyosarcoma is another tumor that
should be suspected in any child with progressive proptosis.
Question 24 of 130
A 10-year-old boy sustained a right upper eyelid laceration after falling from his bicycle. The laceration measures 15 mm and
extends from the eyelid margin to above the eyelid crease. There is an avulsed avascular section of the laceration
superiorly. The results of his ocular examination are normal except for marked swelling of the eyelid. What would the least
appropriate action in the management of this case be?

Check the tetanus status

Ddiscard the avulsed tissue

Repair the eyelid margin

Check the status of the levator muscle


Please select an answer
Feedback: Eyelid lacerations often appear more destructive than they actually are. Proper cleaning of the laceration is
important. A complete eye examination is required to detect ocular injuries, which are often associated with eyelid
lacerations. The tetanus status is important, especially if the exact history of the injury is unknown. Repair of eyelid
lacerations requires a good working knowledge of eyelid anatomy, since repair of deeper structures, such as a dysfunctional
levator muscle, is often required. In deep lacerations, a foreign body should be searched for and removed if present. Eyelid
margin repair is critical, since it will affect the eyelid contour and appearance. In general, it is best to avoid discarding tissue,
since the facial region ha a good blood supply and the survival rate of reimplanted tissue is excellent.

Question 25 of 130
A 24-year-old man was involved in an automobile accident in which his face struck the dashboard. He denies any loss of
consciousness, but says his vision is blurred in the right eye. Visual acuity is 20/30 OD and 20/15 OS. External examination
shows periocular swelling and ecchymosis of the right eye. Motility examination shows moderate restriction to up gaze of the
right eye and normal versions of the left eye. Biomicroscopy and dilated fundus examinations are normal. What is the most
common location of an orbital blowout fracture?

The orbital floor medial to the infraorbital nerve

The orbital aspect of the zygoma

The orbital aspect of the maxilla and inferior orbital rim

The orbital floor lateral to the infraorbital nerve


Please select an answer
Feedback: A blowout fracture is a fracture of the orbital floor while the inferior orbital rim remains intact. The most common
location of this type of fracture is the posteromedial orbital floor, which is medial to the infraorbital nerve. The inferior orbital
rim and the adjacent anterior portion of the floor are stronger by comparison. The floor is composed primarily of the orbital
plate of the maxilla, which is thinnest on the medial side of the infraorbital sulcus. Most of the medial orbital wall consists of
the very thin lamina papyracea of the ethmoid bone. The lateral orbital wall consists of the zygomatic bone and the greater
wing of the sphenoid and is the strongest of the orbital walls; a blowout fracture rarely occurs in this area.
Question 26 of 130
What is the least likely ocular complication from endoscopic sinus surgery?

Diplopia

Blindness

Tearing

Ptosis
Please select an answer
Feedback: Functional endoscopic sinus surgery is becoming increasingly popular. The availability of endoscopic
instruments has made this surgery useful in the diagnosis and treatment of sinus disease. The close proximity of the
paranasal sinuses and the orbit has led to a number of complications with the use of these endoscopes. Damage to the
nasolacrimal duct may occur in attempting to enlarge the natural ostium into the maxillary sinus. Blindness may occur after
inadvertent entry into the orbit and resultant orbital hemorrhage or direct optic nerve damage. Orbital entry may also lead to
damage of the extraocular muscles, such as the medial rectus muscle, and diplopia. The location of the levator muscle in
the superior orbit makes ptosis from this surgical technique unlikely.

Question 27 of 130
A 56-year-old man complains of an aching sensation around his left eye that has lasted for 6 weeks. The discomfort
increases on upgaze. One week ago, he noted blurred vision in the left eye and a low-grade fever. His visual acuity is 20/20
OD and 20/40 OS. The patient has 3 mm of proptosis in the left eye and mild erythema and tenderness around the left
eyelid. Results of biomicroscopy and dilated fundus examination are normal. What is the most helpful diagnostic test for this
patient?

CT scan of the orbits

Complete blood count

Thyroid function tests

Skull films
Please select an answer
Feedback: In this case, the most helpful tool for evaluating proptosis and motility abnormalities would be a CT scan. The
differential diagnosis can be rapidly narrowed to orbital tumor, orbital cellulitis, orbital pseudotumor, and thyroid-related
orbitopathy. A complete blood count is useful in evaluating the patient's fever. Orbital cellulitis or an orbital abscess may
result in an elevated white blood cell count. Orbital pseudotumor may occasionally be associated with an increased
eosinophil count, but affected patients usually have a normal complete blood count. Unilateral proptosis and a motility
abnormality in any patient should raise the concern of thyroid disease, especially in the presence of eyelid abnormalities
such as lid retraction or lid lag. In evaluating an orbital process, skull films are unlikely to provide useful information that
would not immediately require confirmation and further detail from a CT scan or magnetic resonance imaging (MRI).
Question 28 of 130
You are called to consult on a patient who is incapacitated from a cerebrovascular accident and lives in a nursing home. He
continues to have poorly controlled hypertension and remains on oral Coumadin (warfarin). His internist states that the
patient is a poor surgical candidate because of his cardiovascular status. The patient continually complains of foreign-body
sensation and discharge in one eye. Which of the following procedures is most appropriate in this setting?

rRattachment of the capsulopalpebral fascia

A lateral tarsal strip procedure

Rotational sutures (Quickert sutures)

Tarsal wedge excision


Please select an answer
Feedback: In a debilitated or anticoagulated patient with entropion, placement of rotational sutures at the bedside with local
anesthetic provides a lower anesthetic and surgical risk than more complex procedures requiring higher levels of
anesthesia. The other procedures listed require a greater level of preparation and instrumentation and present an increased
risk of postoperative hemorrhage in this anticoagulated patient.

Question 29 of 130
A patient with new onset ocular myasthenia gravis should have a chest CT scan done to look for what associated condition?

Thymoma

Sarcoid

Apical lung tumor (Pancoast's tumor)

Thyroid disease
Please select an answer
Feedback: Thymoma may occur in up to 10% of patients with myasthenia gravis; consequently, a patient with new-onset
myasthenia gravis should be evaluated for the existence of this tumor. Thymectomy would be the appropriate treatment if a
mass is diagnosed. Graves' disease is seen in approximately 5% of these patients and can be diagnosed by an orbital CT
scan in conjunction with a clinical examination. An apical lung tumor could be associated with Horner's syndrome; however,
this patient did not present with symptoms suggestive of this syndrome. Also, there is no association between Horner's
syndrome and myasthenia gravis. Sarcoidosis would most likely present with infiltrative or fibrotic pulmonary changes seen
on chest x-ray, splenomegaly, and lymph node infiltration.
Question 30 of 130
Which of the following does not indicate a poor prognosis for a sebaceous cell adenocarcinoma lesion?

Duration of symptoms less than 6 months

Infiltrative growth pattern

Moderate or poor differentiation

Lymphatic or vascular invasion


Please select an answer
Feedback: Nonocular sebaceous carcinomas exhibit a relatively benign course and rarely metastasize. In contrast, eyelid
sebaceous carcinomas frequently exhibit metastasis and have a poor prognosis. Poor prognostic factors include duration of
symptoms longer than 6 months, size greater than 10 mm, an infiltrative growth pattern, moderate or poor differentiation,
lymphatic or vascular invasion, orbital invasion, and multicentric growth.

Question 31 of 130
Which of the following is least useful in the evaluation of a patient with acquired ptosis?

Interpalpebral fissures

Frontalis muscle excursion

Levator muscle function

Margin-reflex distance
Please select an answer
Feedback: The evaluation of a patient with ptosis is very important in determining proper management of the condition. The
interpalpebral fissure is the distance between the upper and lower eyelid margins. The margin-reflex distance (MRD) is the
distance between the upper eyelid margin and the corneal light reflex in primary gaze. This allows for an accurate measure-
ment of ptosis when the lower eyelid margin is not in its normal position at the limbus. A positive MRD is the number of
millimeters above the light reflex, whereas a negative MRD would be the number of millimeters below the corneal light
reflex. Ptosis is present when the MRD is 2 mm or less. The MRD allows for comparison of the position of each upper eyelid
and the amount of ptosis in unilateral cases. The levator function is an important measurement in determining the type of
ptosis surgery to be performed. This is done by measuring the distance the upper eyelid moves between extreme down
gaze and extreme up gaze. Acquired ptosis usually has normal levator function (12 mm or more), whereas congenital ptosis
will demonstrate a diminished levator function. The eyelid crease is usually higher than normal in acquired ptosis and absent
or faint in congenital ptosis. The influence of the frontalis muscle is eliminated during ptosis evaluation by immobilizing this
muscle with the examiner's thumb on the brow while measuring the levator function. Frontalis muscle contraction is
commonly seen as a compensatory mechanism in patients with ptosis. The degree of frontalis muscle excursion is of limited
value, however, in the assessment of patients with acquired ptosis.
Question 32 of 130
Which of the following orbital diseases is least likely to improve with corticosteroids?

Orbital mucocele

Thyroid-related orbitopathy

Orbital pseudotumor

Orbital lymphoma
Please select an answer
Feedback: It is important to recognize that many orbital processes can respond initially to oral or intravenous
corticosteroids. These include lymphomatous or inflammatory processes such as thyroid-related orbitopathy and orbital
pseudotumor. Although pseudotumor has been described as the most likely diagnosis in this scenario, the physician should
remain concerned about the possibility of these other diagnoses. An orbital mucocele is related to chronic inspissation of the
sinus from blockage of its normal drainage ostium. A diagnosis of mucocele is suspected by findings on CT scan. Mucoceles
usually do not respond to corticosteroids, but would require surgical drainage.

Question 33 of 130
What is the best way to distinguish a restrictive motility disorder from a paretic disorder after a blowout fracture?

Alternate prism cover testing

Goldmann single binocular visual fields

Motility examination of the cardinal positions of gaze

Forced-duction testing
Please select an answer
Feedback: Alternate prism cover tests, Goldmann single binocular fields, and a careful motility examination are all good
ways to identify or quantify ocular misalignment, but will not distinguish paralytic from restrictive cases. Forced-duction
testing is the best clinical tool to distinguish these two entities and should be done several days after the injury. Edema
and/or a hematoma may cause initial forced ductions to appear positive and give a false impression that entrapment exists.
After topical anesthesia is obtained with tetracaine or cocaine, the episcleral tissues at the inferior limbus are grasped with
toothed forceps. The eye is then rotated in the desired direction, and comparisons can be made with the uninjured side. In
cases of paralytic strabismus, the forced ductions are free, unlike cases of restrictive strabismus.
Question 34 of 130
Which of the following CT findings is not commonly seen with thyroid-related orbitopathy?

Sparing of extraocular muscle tendons

Involvement of extraocular muscle tendons

Fusiform extraocular muscle involvement

Bilateral extraocular muscle involvement


Please select an answer
Feedback: Thyroid-related orbitopathy is an immunologically related inflammatory condition that affects orbital tissues,
particularly the extraocular muscles. The most common cause for unilateral orbital proptosis in an adult is thyroid-related
orbitopathy. Even though the clinical finding may indicate a unilateral problem, a CT scan or MRI often show bilalateral
disease. The CT findings consist of fusiform enlargement of the extraocular muscle, commonly the inferior and medial rectus
muscles, with sparing of the tendons.

Question 35 of 130
Which of the following is not a potential advantage of MRI over CT scanning?

MRI does not expose the patient to radiation.

MRI is unaffected by motion artifact.

MRI can generate high quality axial, coronal, and sagittal image without repositioning the patient.

MRI allows for better valuation of lesions that extend from the orbit to the cranium.
Please select an answer
Feedback: Magnetic resonance imaging (MRI) is a valuable modality in the evaluation of orbital disease. Unlike
computerized tomography (CT), MRI doe not expose the patient to radiation. The image is generated by exposing body
tissue placed in a strong magnetic field to a radio frequency pulse. MRI does require more time than a CT scan and is more
sensitive to motion artifact. MRI may give superior detail of orbital soft tissue and is superior in evaluating processes that
extend from the orbit into the cranium. It is not necessary to adjust the position of the patient to obtain axial, coronal, sagittal,
parasagittal, or oblique images with a spatially coordinated MRI scan. A CT scan image may be reconstructed in off-axis
planes; however, the quality of the image is suboptimal. Even with these advantages, MRI has not replaced CT scanning in
the evaluation of orbital disease. The majority of orbital processes can be adequately evaluated with a CT scan. CT scan
provide bony imaging, important for orbital surgery, which is not present on MRI scan. CT scanning and MRI offer distinct
advantages, and the appropriate election should be based on the clinical situation.

Question 36 of 130
Which of the following is not a common sign of an orbital blowout fracture?

Tenderness over the zygomaticofrontal suture

Infraorbital hypoesthesia

Restriction of supraduction

Enophthalmos
Please select an answer
Feedback: Numbness of the ipsilateral cheek, upper lip, and upper teeth is a classic finding of a blowout fracture. This
hypoesthesia is a result of edema or direct injury to the infraorbital nerve as it courses through the infraorbital canal in the
vicinity of the fracture site. Restrictive strabismus is present in 40% to 63% of patients with isolated floor fractures.
Enophthalmos may be present immediately or may develop later; it can be quantified by using an exophthalmometer. Acute
enophthalmos is due to loss of orbital tissue into the maxillary or ethmoid sinus. Late-onset enophthalmos results from
atrophy and/or contraction of orbital tissues. Other signs that suggest enophthalmos are a deepening of the superior sulcus
or a narrowed palpebral fissure. Tenderness over the zygomatkofrontal suture is not characteristic of an isolated blowout
fracture but is a common finding in a zygomaticomaxillary (tripod) fracture.

Question 37 of 130
During routine examination of a patient's inferior cul-de-sac, a subconjunctival lympho-proliferative lesion is observed. The
patient is unaware of this lesion and is reportedly in good health. The results of the remainder of the ocular examination are
normal. A biopsy is done. What would the least useful test performed on this biopsy be?

Permanent sections

Culture and sensitivity

Cell-surface markers

Electron microscopy
Please select an answer
Feedback: Lesions suspected to be lymphomatous in nature should undergo biopsy. One should consult preoperatively with
the pathologist to discuss the proper method of handling the tissue. The specimen should be divided into three sections.
One section should go for permanent sections and be stored in formalin. The second section should be submitted on a
saline wet gauze or sponge for immunopathologic studies to determine cell-surface markers. The third section can be stored
in glutaraldehyde for electron microscopic studies if needed. Electron microscopic studies can aid in differentiating some of
these tumors when there is confusion, but they are needed less commonly with current diagnostic testing. The clinical
picture is not consistent with an infectious process, so culture would be the least useful test in the management of this
patient.

Question 38 of 130
You are asked to evaluate a 20-year-old man in the emergency room who has been hit over the left brow with a hockey
stick. The visual acuity is 20/20 OD and 20/200 OS. A large hematoma is forming in the left upper eyelid, and the eyelid is
tense. A left relative afferent pupillary defect is present. Anterior segment examination reveals a left subconjunctival
hemorrhage with a microscopic hyphema. It is not possible to measure the intraocular pressure in the left eye; however, the
left globe is tense to digital palpation. Dilated fundus examination of the left eye reveals peripheral retinal edema and
pulsation of the central retinal artery. What would the most appropriate emergent management be?

Order an emergent CT scan

Perform a paracentesis

Perform a canthotomy and cantholysis

Begin intravenous corticosteroids


Please select an answer
Feedback: The clinical features of a tense upper eyelid, increased intraocular pressure, and pulsation of the central retinal
artery are consistent with an acute orbital compartment syndrome. The orbital pressure needs to be lowered immediately.
The most expeditious method is a canthotomy and cantholysis. Both the superior and inferior limbs of the lateral canthal ten-
don should be released for maximal decompression. Following this procedure, the fundus should be reexamined to make
sure that arterial flow has been restored. The relative afferent pupillary defect may be due to a fracture of the optic canal;
however, treatment should be directed toward lowering the intraorbital pressure before a CT scan is obtained. A
paracentesis may lower the intraocular pressure transiently; however, this will not adequately address the compartment
syndrome. Surgical or medical (corticosteroids) orbital decompression should be considered if the relative afferent pupillary
defect does not resolve. This would indicate that a component of compressive optic neuropathy persists and should be
treated.

Question 39 of 130
A previously healthy 6-year-old child presents with proptosis of the left eye. Family photographs reveal some prominence of
the eye for the past year. One week prior to presentation, the child had a seizure of undetermined cause. Fundus
examination reveals choroidal folds OS. Which one of the following diagnostic tests is least useful in this case?

MRI

CT scan

Echography

Fluorescein angiography
Please select an answer
Feedback: The findings of proptosis and choroidal folds suggest the presence of a mass behind the eye. A CT scan, MRI,
or echographic examination of the orbit would all be reasonable imaging modalities to evaluate this patient. The occurrenc of
a seizure in this child raises the suspicion of intracranial involvement. MRI is often superior to CT scan in imaging lesions
that involve both the orbit and intracranial structure, although CT scanning is usually preferred to MRI in lesions confined to
the orbit. Fluorescein angiography is not likely to add any useful information regarding the cause of proptosis of this patient.

Question 40 of 130
A 75-year-old woman complains of restriction of her upper field of vision and difficulty reading when looking down. She
denies any discomfort, epiphora, or diplopia. Her vision is J1+ OU through her well-positioned bifocal segments. A basic tear
secretion test is normal. Examination shows an eyelid malposition. What is the most likely diagnosis?

Entropion

Dermatochalasis

Involutional ptosis

Ectropion
Please select an answer
Feedback: Involutional ptosis results from a dehiscence or disinsertion of the levator aponeurosis. This ptosis is
accentuated on down gaze in many patients, resulting in further closure of the ptotic eyelid. These patients not only lose the
superior visual field in primary gaze, but also note visual field impairment in down gaze, since the eyelid rests in a lower
position. Some patients with normal palpebral fissures in primary gaze have ptosis with down gaze. Entropion could
conceivably cause difficulty in reading; however, ocular discomfort or pooling of tears would be expected. Ectropion and
dermatochalasis, unless excessive, are unlikely to cause problems with reading.
Question 41 of 130
What are the 2 most commonly affected rectus muscles in thyroid eye disease?

Superior and inferior

Superior and medial

Medial and lateral

Inferior and medial


Please select an answer
Feedback: The inferior rectus is most commonly involved in thyroid eye disease, followed by the medial rectus and superior
rectus.

Question 42 of 130
What type of orbital blow-out fracture requires urgent referral?

medial wall fractures

fractures involving more than 4 mm of displacement

small symptomatic fractures in patients younger than age 18 years

fractures associated with a blood-filled maxillary sinus


Please select an answer
Feedback: Patients younger than 18 years are at greater risk of small trap-door floor fractures that entrap orbital tissues and
lead to rapid infarction and permanent extraocular muscle restriction.

Question 43 of 130
What is the best study to rule out organic orbital foreign bodies?

Magnetic resonance imaging

A dowsing rod

Plain films

Computerized tomography
Please select an answer
Feedback: Magnetic resonance imaging is good for organic foreign bodies; computerized tomography and plain films are
adequate for metal foreign bodies. Controlled studies of dowsing have been disappointing.
Question 44 of 130
Which condition is closely associated with thyroid eye disease?

Eczematous eyelid

Parinaud's syndrome

Myotonic dystrophy

Myasthenia gravis
Please select an answer
Feedback: Thyroid eye disease may be associated with other autoimmune manifestations such as myasthenia gravis. Both
Parinaud's dorsal midbrain syndrome and excema may cause eyelid retraction, but are not closely related to thyroid eye
disease. Myotonic dystrophy causes ptosis and is not closely related to thyroid eye disease.

Question 45 of 130
What is the most common organism implicated in dacryocystitis?

non-septate fungi

gram-positive bacteria

septate fungi

gram-negative bacteria
Please select an answer
Feedback: Gram-positive organisms are most common in dacryocystitis. Consider gram-negative organisms in patients with
diabetes mellitus, immunocompromised patients, or nursing home residents if there is no response to initial treatment.
Fungal infection would be a rare cause of dacryocystitis.

Question 46 of 130
What is the study of choice for the evaluation of fractures in acute orbital trauma?

Orbital ultrasound

Computerized tomography

Magnetic resonance imaging

Nerve conduction
Please select an answer
Feedback: Computerized tomography is superior for evaluation of bone disorders. Magnetic resonance imaging and orbital
ultrasound would be of less utility and nerve conduction is not commonly done in the setting of acute orbital trauma.
Question 47 of 130
Blood-tinged tears should prompt what treatment?

Balloon dacryoplasty

Biopsy of lacrimal sac

Probing and tube placement

Dacryocystorhinostomy
Please select an answer
Feedback: Malignancy should be considered in patients with blood-tinged tears. Biopsy is necessary to establish the
diagnosis. Probing or balloon dacryoplasty would fail to establish the diagnosis, and dacryocystorhinostomy could spread
the tumor.

Question 48 of 130
Which one of the following is likely to occur, with respect to the epithelium, of the transplanted tissue of a hard palate graft?

It will maintain some form of keratinization (orthokeratosis and/or parakeratosis)

It will remain fully keratinized

It will convert from keratinized to nonkeratinized

All epithelium will be lost


Please select an answer
Feedback: Full thickness mucosal grafts typically maintain some form of keratinization following transplantation to the ocular
surface.

Question 49 of 130
Chronic unilateral blepharoconjunctivitis is commonly a presenting sign of which one of the following?

Squamous cell carcinoma

Basal cell carcinoma

Cutaneous melanoma

Sebaceous carcinoma
Please select an answer
Feedback: Sebaceous cell carcinoma commonly presents as a unilateral blepharoconjunctivitis. The other entities would be
much less likely to involve the conjunctival surface of the eyelid to a significant degree and usually present differently.
Question 50 of 130

A patient with acute dacryocystitis, reflux of pus from the canaliculi, and preseptal cellulitis should be treated with which of
the following?

Immediate dacryocystorhinostomy

Massage

Systemic antibiotics

Probing and irrigation for diagnosis confirmation


Please select an answer
Feedback: A patient with obvious dacryocystitis should not be probed. Probing would be painful and not provide additional
information. Antibiotics will help inflammation subside prior to dacryocystorhinostomy. Massage may have some role in
chronic dacryocystitis.

Question 51 of 130
How does lower eyelid retractor repair for involutional entropion of the lower eyelid work by?

Reattaching the capsulopalpebral fascia to the tarsus

Shortening the septum

Repairing cicatricial changes

Horizontally shortening the orbicularis


Please select an answer
Feedback: The three currently accepted etiologic mechanisms for entropion are canthal tendon laxity, dehiscence of the
capsulopalpebral fascia, and over-ride of the preseptal orbicularis pushing the lid margin inward. Some (but not all) surgical
techniques for entropion address all of these mechanisms.

Question 52 of 130
A 75-year-old man has acute dacryocystitis. Appropriate management consists of which of the following procedures?

Irrigation of the lacrimal sac

Oral antibiotics

Canalicular probing to relieve infection pressure

Oral steroids
Please select an answer
Feedback: Acute dacryocystitis typically requires oral antibiotics and dacryocystorhinostomy (DCR). External DCR is
commonly performed after the acute infection has subsided, but endoscopic DCR can be performed urgently to drain the
abscessed lacrimal sac. Irrigation and probing are likely to cause iatrogenic damage to the lacrimal system and should not
be performed acutely.
Question 53 of 130
The myositic form of idiopathic orbital inflammation is associated with which of the following conditions?

Efficacy of systemic steroid therapy

S-shaped deformity of the eyelid

Fusiform enlargement of extraocular muscle involving the tendon

Nodular enlargement of the extraocular muscle belly


Please select an answer
Feedback: The myositic form of idiopathic orbital inflammation often presents with strabismus and less intense pain,
redness, and proptosis than the more common diffuse form. It is more likely to relapse during steroid taper and may require
antimetabolite therapy.

Question 54 of 130
What is the appropriate treatment for acute dacryocystitis with localized abscess?

Irrigation and probing of the lacrimal sytem followed by application of warm compresses

Oral antibiotics and drainage of abscess or immediate dacryocystorhinostomy

Topical antibiotics

Surgical creation of a permanent dacryocutaneous fistula


Please select an answer
Feedback: Dacryocystitis with localized abscess is best treated with systemic antibiotics and prompt drainage of the
abscess. Dacryocystorhinostomy surgery will accomplish drainage, but often cannot be performed in a timely fashion, and
can be technically challenging due to local cellulitis and the possibility of rupture and fistulization. Probing is likely to damage
the inflamed canaliculi or the valve of Rosenmuller, and topical antibiotics are inadequate. Dacryocutaneous fistula is an
undesired outcome.

Question 55 of 130
Which of the following statements most accurately describes the behavior and management of congenital nevi?

The risk of melanoma is directly related to the size of congenital nevi.

Small congenital nevi do not need to be followed.

Biopsy of congenital nevi is contraindicated.

Large congenital nevi require complete surgical excision.


Please select an answer
Feedback: Giant congenital nevi (>20 cm diameter) have a risk of transformation into melanoma of 5-7% by age 60. Smaller
nevi have an undetermined risk of transformation and can be observed or biopsied.
Question 56 of 130
What is a common sign of a malignant lymphoproliferative lesion?

Firm nodular anterior orbital mass

Painful proptosis

Vision loss

Madarosis
Please select an answer
Feedback: Common malignant lymphoproliferative lesions in the orbit include non-Hodgkins lymphoma and myeloma and
these commonly present with insidious subacute mass effect rather than pain and vision loss, as can be seen with idiopathic
orbital inflammation and some epithelial malignancies.

Question 57 of 130
A patient presents with lentigo maligna involving the majority of the lower eyelid. What is the most appropriate management
option?

Observation for thickening of the lesion

Radiation

Cryotherapy

Surgical excision with pathologic confirmation and delayed reconstruction


Please select an answer
Feedback: Lentigo maligna, or superficial melanoma, requires complete eradication of the tumor and excision is the most
certain method to achieve this. Cryo and Radiation each may be incomplete. The melanin in the this tumor prevents frozen
section analysis and instead requires bleaching with pathologic exam the next day and hence delayed reconstruction until
the margins are clear.

Question 58 of 130
Recurrent unilateral, or bilateral, eyelid swelling in a younger patient is suggestive of which of the following diagnoses?

Hemifacial spasm

Gorlin's syndrome

Dermatochalasis

Blepharochalasis
Please select an answer
Feedback: Blepharochalasis is an idiopathic disorder causing recurrent eyelid edema in younger patients, with gradual
development of dermatochalasis and ptosis.
Question 59 of 130
A recurrent squamous cell carcinoma is excised from the medial canthus. Which of the following reconstructive techniques
should be avoided to prevent detection of a deep delayed recurrence?

Midforehead rotational flap

Full thickness skin graft from the retroauricular area

Full thickness skin graft from the upper eyelid

Undermining with direct closure


Please select an answer
Feedback: Recurrent squamous cell carcinoma can be difficult to completely excise from the medial canthus, and deep
delayed recurrence is not uncommon. Thick flaps (e.g. forehead) may hide recurrences and hence may not be the best
option for reconstruction.

Question 60 of 130
Which of the following is the most common indication for repair of medial orbital wall fractures?

Epiphora

Entrapped inferior rectus muscle

Sinusitis

Entrapped medial rectus muscles


Please select an answer
Feedback: Medial orbital wall fractures often cause enophthalmos and/or medial rectus entrapment with restrictive
strabismus, both of which are indications for surgical repair.

Question 61 of 130
What is the most common complaint following successful correction of paralytic ectropion?

Consecutive entropion

Prolonged chemosis

Persistent epiphora

Overelevation of the lateral canthal angle


Please select an answer
Feedback: Paralytic ectropion is commonly accompanied by failure of the lacrimal pump and epiphora. Successful
correction of the eyelid malposition typically does not restore lacrimal pump function since the orbicularis does not regain
normal tone.
Question 62 of 130
Which of the following is a contraindication to Muller's muscle conjunctival resection?

Acquired aponeurogenic ptosis

Post-cataract extraction ptosis

No eyelid position change following instillation of topical phenylephrine

Mild congenital ptosis


Please select an answer
Feedback: Muller's muscle conjunctival resection can be employed in the correction of blepharoptosis from a variety of
causes. It is most predictably useful in patients who exhibit an elevation of the affected eyelid(s) following the instillation of
phenylephrine eyedrops.

Question 63 of 130
Which of the following statements describes how to differentiate a compound from a junctional nevus?

Compound nevi are larger

Junctional nevi are darker and macular, or thinly papular, while compound nevi are lighter and elevated compared to
uninvolved surrounding skin

Junctional nevi show melanocytes in the superficial dermis

Junctional nevi are more dome-shaped


Please select an answer
Feedback: Acquired nevi include junctional, compound, and intradermal subtypes. Others include dysplastic nevi, Spitz
nevi, and blue nevi. Correct diagnosis can help establish risk of malignant melanoma.

Question 64 of 130
If a patient has a ruptured globe in addition to a symptomatic blowout fracture, what should the surgeon do?

Repair both injuries at the same time

Repair the ruptured globe immediately and delay the floor exploration for 2 to 4 weeks

Wait 48 hours and then repair both injuries at the same time

Repair the ruptured globe immediately and delay the floor exploration for 4 months
Please select an answer
Feedback: The repair of a ruptured globe takes priority over a blowout fracture. It is not advisable to undertake surgical
repair of both injuries at the same time. A blowout repair requires considerable tractional forces, which might worsen the
anterior or posterior segment injury. A delay of 2 to 4 weeks should have no adverse effect on the eventual outcome of a
blowout repair.
Question 65 of 130
A patient has sustained blunt facial and orbital trauma in a motor vehicle accident. The patient has limited supraduction and
marked upper eyelid edema and ptosis. A CT scan shows a frontal sinus fracture, orbital roof fracture, and
pneumocephalus. What would the appropriate course of action be?

Neurosurgical consultation to rule out intracranial injury

Valsalva to evaluate brain-orbit interface

Forced-duction testing to rule out superior rectus muscle entrapment

Superior orbitotomy to rule out a levator laceration


Please select an answer
Feedback: An orbital roof fracture is a potentially life-threatening injury. Intracranial air is a clue that a more extensive injury
may be present. Immediate neurosurgical consultation is indicated to rule out intracranial injury or dural tear with leakage of
cerebrospinal fluid. Meningitis and brain abscess are devastating complications of this type of orbital fracture. If a bony roof
fragment is not readily apparent and impinging on the superior rectus levator complex, then waiting for resolution of edema
is indicated. Traumatic ptosis often resolves spontaneously. In the absence of worsening proptosis, a carotid cavernous
sinus fistula is unlikely.

Question 66 of 130
When performing a lateral tarsal strip for horizontal lid laxity of the lower lid, what is the correct placement of the lateral
canthus?

2 millimeters lower than the medial canthus

Outside the lateral orbital rim

At Lookwood's tubercle

2 millimeters above the medial canthus


Please select an answer
Feedback: The normal lateral canthal position is approximately 2 mm higher than the ipsilateral medial canthus, with the
eyelid directed inside the orbital rim toward Whitnall's tubercle.

Question 67 of 130
What is the pathophysiologic mechanism underlying epiblepharon?

Laxity of the tarsal plate

Abnormal attachment of the orbital septum

Redundancy of skin and orbicularis oculi muscle

Laxity of the canthal tendons


Please select an answer
Feedback: The most likely mechanism for epiblepharon is an excess of pretarsal skin and orbicularis oculi muscle at the
lower eyelid margin. The elevation of these tissues near the eyelid margin forces an upward and inward rotation of the lower
eyelashes. A common result of this rotation is contact between the lower eyelashes and the cornea or inferior bulbar
conjunctiva. Laxity of the tarsal plate may be seen as an atrophic change in adults. Laxity of the canthal tendons is an
involutional change and would not be expected in children. Abnormal orbital septal attachments may occur as a
postoperative or posttraumatic complication, but are an unlikely cause of eyelash malposition.

Question 68 of 130
Ectropion and loss of eyelashes should alert one to the possibility of which one of the following?

Facial nerve (VII) palsy

Chronic eyelid webbing

Involutional ectropion

Malignancy
Please select an answer
Feedback: Ectropion combined with alteration of eyelid anatomy, or loss of eyelashes, should suggest the possibility of
malignancy and need for a biopsy. Loss of eyelashes is not typical of involutional changes, facial palsy, or eyelid rubbing.

Question 69 of 130
Excisional biopsy is a useful treatment modality in which of the following?

Lattice corneal dystrophy

Ocular cicatricial pemphigoid

Nodular scleritis

Conjunctival intraepithelial neoplasia


Please select an answer
Feedback: Conjunctival intraepithelial neoplasia is best managed with biopsy to evaluate histology. Instead, biopsy of
nodular scleritis and OCP would both be expect to increase local inflammation without providing useful pathology. Biopsy of
lattice corneal dystrophy is likely to cause vision-threatening scarring and also not provide pathology that will help in clinical
decision making.

Question 70 of 130
Which of the following features of nevi is associated with an increased risk of malignancy?

The nevus has lost its color

The nevus has an irregular border

The nevus is growing

The nevus is noted congenitally


Please select an answer
Feedback: Congenital nevi have a 5-10% lifetime risk of transformation to melanoma, according to numerous published
series. The other choices can be observed among benign nevi.
Question 71 of 130
What is the optimal procedure for analysis of the bony orbit?

Magnetic resonance imaging

Contrast enhanced plain X-ray films

Ultrasound

Computerized tomography
Please select an answer
Feedback: Computer tomography is the optimal imaging technology for examining the bones of the orbit, such as in cases
of trauma to the bony orbit.

Question 72 of 130
Computerized tomography has demonstrated an orbital bone mass to have a "ground glass" appearance. What systemic
involvement should be ruled out?

Generalized muscle weakness

Visceral cancer

Thyroid disease

Endocrine abnormality
Please select an answer
Feedback: Fibrous dysplasia may be associated with cutaneous pigmentation and endocrine disorders in Albright
syndrome.

Question 73 of 130
You have removed a medial canthal lesion which is read as basal cell carcinoma with morpheaform characteristics. The
pathologist confirms the margins are negative in four quadrants (0°, 15°, 30°, 45°). What is the optimum next step?

Adjunctive cryotherapy

Excision with margin control because of the aggressive nature of the tumor

Adjunctive alkylating agents

Observation with return in one year


Please select an answer
Feedback: Morpheaform basal cell carcinomas are aggressive and require complete margin control.
Question 74 of 130
Biopsy of a broad area of pigmentation of the eyelid has been read as lentigo maligna. What is the treatment of choice?

Complete excision with adequate surgical margins

Map biopsies looking for localized invasion

Cryotherapy to the broad area

Close observation
Please select an answer
Feedback: Lentigo maligna progresses to vertically invasive melanoma in nearly half of patients. Complete excision is
indicated.

Question 75 of 130
What is the treatment of choice for keratocanthoma?

Cryotherapy

Corticosteroid injection

Observation

Incisional biopsy followed by complete surgical excision


Please select an answer
Feedback: Many authors now regard keratocanthoma as low-grade or well-differentiated squamous cell carcinoma.
Incisional biopsy followed by complete surgical excision is recommended

Question 76 of 130
What is sebaceous cell carcinoma's response to radiation therapy?

Excellent stand-alone therapy

Very susceptible when used as an adjunct to surgery

Responsive when combined with photodynamic agents

Relatively radioresistant
Please select an answer
Feedback: Sebaceous cell carcinoma is relatively radioresistant.
Question 77 of 130
A suspected upper eyelid chalazion in a 68-year-old patient demonstrates surrounding palpebral conjunctival inflammation,
raising concern about sebacaeous cell carcinoma. What is the optimum next step?

Sentinal lymph node evaluation

Full thickness biopsy and conjunctive map biopsies

Shave biopsy

Corticosteroid injection and curretage


Please select an answer
Feedback: Diagnosis of sebaceous cell carcinoma frequently requires full-thickness examination of the eyelid, and
conjunctival biopsies will help define the extent of the disease.

Question 78 of 130
What is the role of cryotherapy in the treatment of eyelid melanoma?

Possibly useful in conjunctival melanoma, but not in skin

Recommended for both skin and conjunctiva

Useful for skin, not conjunctiva

Not useful for skin or conjunctiva


Please select an answer
Feedback: Some authors recommend cryotherapy as adjunctive treatment for conjunctival melanomas, but it is not
recommended for cutaneous melanoma.

Question 79 of 130
What is the optimal imaging technique for a posterior optic nerve glioma?

Ultrasound

Magnetic resonance imaging

Plain X-ray films

Computerized tomography
Please select an answer
Feedback: MRI provides the best tissue contrast of structure in the orbital apex and intracanalicular portion of the optic
nerve.
Question 80 of 130
What is the most important predicator for recurrence and survival in patients with eyelid skin lesion?

Excision margins

Tumor thickness

Diameter

Geographic Location on eyelid


Please select an answer
Feedback: Breslow thickness is the most important predictor of recurrence and survival in patients with eyelid skin
melanoma.

Question 81 of 130
What is the mainstay of treatment for rhabdomyoscaroma?

Chemotherapy and radiation

Immunotherapy

Surgery

Observation
Please select an answer
Feedback: The mainstay for treatment of rhabdomysarcoms is chemotherapy and radiation. Surgery is used in some
circumstances for debulking in combination with other therapies, and immunotherapy is not a first-line treatment.

Question 82 of 130
When passing a probe down the cannalicular portion of the lacrimal excretory system, at what length does the tip of the
cannula enter the lacrimal sac?

12-16 mm

16-20 mm

8-12 mm

4-8 mm
Please select an answer
Feedback: The canalicular system is 8-12 mm in length. Usually the canaliculi combine to form a common canaliculus.
Question 83 of 130
What is the most common eyelid condition associated with the use of topical latanoprost?

Depigmentation of the iris and periocular skin

Eyelid margin necrosis

Hyperpigmentation of periocular skin and eyelid-margin hyperemia

Severe hirsutism of periocular skin


Please select an answer
Feedback: Side effects of latanoprost include hypertrichosis, increased pigmentation of eyelashes and eyelids,
hyperpigmentation of the iris, and mild conjunctive hyperemia.

Question 84 of 130

When planning to surgically repair a congenital ptosis in a patient, what is the most important pre-operative assessment?

Intercanthal distance

Levator function

Amount of ptosis in millimeters

Eyelid crease height

Please select an answer


Feedback: The surgical technique used for congenital ptosis depends primarily on levator function; patients with poor
levator function often require some form of frontalis sling. The severity (in mm) of ptosis is secondary. Patients with marked
ptosis and good levator function can still benefit from levator surgery

Question 85 of 130
What would be the preferred management to treat a patient with membranous, congenital, nasolacrimal duct obstruction,
and stenosis of both upper and lower canaliculi?

Bicanalicular intubation of the nasolacrimal duct

Bicanalicular intubation with dacrocystorhinostomy

Monocanalicular intubation

Bicanalicular ring intubation with pigtail probe


Please select an answer
Feedback:
Constriction or stenosis of the canaliculi is treated with dialation and silicone intubation. Monocanalicular intubation would
treat only one canaliculus; bi-canalicular ring intubation would not address the membranous nasolacrimal duct obstruction;
dacryocystorhinostomy is not indicated for simple membranous obstruction of the nasolacrimal duct.

Question 86 of 130
For which orbital disease can increased orbital fat volume be a primary radiographic finding?

Orbital myositis

Thyroid orbitopathy

Sarcoidosis

Wegner's granulomatosis
Please select an answer
Feedback: Enlarged extraocular muscles with tendon sparing is a common feature of thyroid orbitopathy. Another
radiograpic presentation is increased orbital fat volume with or without enlarged muscles.

Question 87 of 130

Arteriovenous fistulas that affect the orbit most commonly develop following what type of trauma?

Orbital blowout fracture

Frontal sinus fracture

Penetrating intacranial trauma

Basal skull fracture


Please select an answer
Feedback: The most common type of arteriovenous fistula that affects the orbit is a carotid cavernous fistula. The
cavernous sinus is venous lumen involved, and the internal carotid artery is the most common source of afterial blood. A
basal skull fracture is the type of injury that most comonly produces an arteriovenous fistula in this location. Spontaneous
fistula formation occurs most often as a degenerative process resulting from system hypertension and atherosclerosis.
Question 88 of 130
A patient with an optic-nerve sheath meningioma confined to the orbit has visual acuity of 20/30. What would be the best
treatment at this stage?

Radiation therapy

Systemic steroid therapy

Surgical resection

Observation
Please select an answer
Feedback: Observation is indicated if vision is minimally affected and no intracranial extension is present. If the tumor is
confined to the orbit and vision loss is significant or progressive, fractionated stereotractic radiotherapy should be
considered. Surgical excision usually results in vision loss, therefore surgery is reserved for patients with severe vision loss
and profound proptosis.

Question 89 of 130
A 6-year-old presents with proptosis and inferior-lateral displacement of the globe. Imaging demonstrates clear sinuses and
a large orbital mass. What diagnostic or therapeutic step should be considered next?

Prompt biopsy with possible frozen section diagnosis, bone marrow biopsy, and lumbar puncture

Attempted aspiration of the mass with empiric antibiotics if aspiration is unsuccessful

Discharge home on oral antibiotics

Treat with intravenous antibiotics for 10 days and reevaluate

Please select an answer


Feedback: Orbital abscesses do not generally occur in isolation, but originate from sinus disease or an eyelid process.
Rhabdomyosarcoma classically presents with downward and lateral displacement of the globe. Preparation for possible
metastatic work-up could help avoid repeated general anesthesia.

Question 90 of 130
When diplopia develops in the setting of traumatic carotid cavernous fistula, what is the most likely pathophysiology?

Compression of the fourth cranial nerve as it exits the brainstem

Compression of the superior rectus muscle within the muscle cone

Damage to the third cranial nerve from elevated intracranial pressure

Compression of the sixth cranial nerve within the cavernous sinus


Please select an answer
Feedback: Carotid cavernous fistulas are associated with increased pressure in the cavernous sinus and can cause
compression of the third and fourth, and most commonly, sixth cranial nerves, with associated muscle palsies.

Question 91 of 130
Unilateral rounding of the medial canthal tendon is a feature of which disorder?

Fracture of the medial wall of the orbit

Connective tissue disease involving the medial canthal tendon

Lacrimal sac tumor

Avulsion of the medial canthal tendon


Please select an answer
Feedback: Medial canthal tendon avulsion should be suspected when there is a rounding of the medial canthus and
acquired telecanthus. Lacrimal sac tumors present as a lacrimal sac mass, frequently extending above the medial canthal
tendon. There are no connective tissue disease that causes rounding of the medial canthus, unless uncontrolled
inflammation results in complete lysis of the medial canthal tendon.

Question 92 of 130
A Quickert suture is most effectively used when repairing what disorder?

Spastic entropion

Distichiasis

Involutional entropion

Cicatricial entropion
Please select an answer
Feedback: Quickert suture repair is considered a temporizing measure with a high recurrence rate, and therefore is most
appropriate for temporary conditions such as acute spastic entropion. A procedure that offers more permanent stabilization
of the eyelid such as retractor repair (for involutional entropion) or grafting (for cicatricial entropion) should be eliminated.
Distachiasis is best managed by epilation or cryopexy.

Question 93 of 130
What is the preferred management of hemangiopericytoma involving the orbit?

Incisional biopsy followed by external radiation

Intralesional steroid injection

Observation

Complete local excision


Please select an answer
Feedback: Hemangiopericytomas are best removed by complete excision. With incomplete excision, they may recur, with
potential malignant degeneration, visual loss, and metastasis. The other responses involve excision or non-excision allowing
for potential malignant degeneration.

Question 94 of 130
A 4-year-old child is referred for a new onset of bilateral epiphora. Examination shows eyelashes on both lower eyelids
rubbing against the inferior cornea. The parents state that an older sibling has the similar symptoms, which resolved without
treatment. What is the most likely diagnosis?

Entropion

Epiblepharon

Euryblepharon

Trichiasis
Please select an answer
Feedback: The history and examination are most consistent with the diagnosis of epiblepharon. Infants may be
asymptomatic because their soft lashes do not irritate the cornea. As children age, the lashes become coarse, causing
ocular irritation and epiphora. The family history of a symptomatic sibling whose epiphora resolved is consistent with the
diagnosis of epiblepharon because the facial features in siblings are often similar. As facial elongation occurs with age,
epiblepharon can spontaneously resolve because excess skin is pulled inferiorly. Entropion results from an inward rotation
of the eyelid margin, which does not occur in epiblepharon. Entropion is unlikely to resolve spontaneously in a child.
Euryblepharon is a congenital widening of the palpebral fissure, usually involving increased vertical widening of the temporal
portion. Trichiasis describes misdirected eyelashes resulting from chronic inflammation of the eyelid margin.

Question 95 of 130
An obese patient has chronic conjunctivitis with upper eyelids that easily evert. What additional feature of this disorder would
you expect to be present?

Tarsal biopsy showing decreased fibrillin

History of hypoglycemia

Follicular conjunctivitis

History of sleep apnea


Please select an answer
Feedback: Floppy eyelid syndrome is characterized by chronic papillary conjunctivitis; easily everted, flaccid upper eyelids;
and nonspecific irritation. Associations include obesity, keratoconus, eyelid rubbing or mechanical pressure, sleep apnea,
and hyperglycemia. A follicular conjunctivitis would not be expected. Decreased fibrillin in a tarsal biopsy is associated with
Marfan's syndrome.
Question 96 of 130
Optic nerve gliomas demonstrate what finding on computer axial tomography (CT) imaging?

Fusiform enlargement of the optic nerve

Hyperostosis of the sphenoid bone

Erosion of the lamina paprycea

Attenuated optic nerve


Please select an answer
Feedback: CT and MRI of optic nerve gliomas show fusiform enlargement of the optic nerve. MRI may show cystic
degeneration. Hyperostosis of the sphenoid bone is characteristic of sphenoid wing meningiomas. Optic nerve gliomas are
associated with enlargement, not attenuation of the nerve. The optic nerve is separated from the lamina paprycea by fat
and extraocular muscles, making erosion unlikely.

Question 97 of 130
What is the test of choice when considering treatment for a carotid cavernous fistula?

Computed tomography

Magnetic resonance imaging

Conventional angiography

Computed tomographic angiography


Please select an answer
Feedback: Conventional angiography is a diagnostic test used when CT and MRI are negative or when treatment is
planned for carotid cavernous fistula. Computerized tomographic angiography, magnetic resonance imaging, and computed
tomography can all miss a carotid cavernous fistula.

Question 98 of 130
What would you expect to find on computerized axial tomography (CT) of a dural sinus fistula?

Extraocular muscle enlargement

Phleboliths

Enlargement of the internal carotid artery

Orbital expansion with Valsalva maneuver


Please select an answer
Feedback: CT scanning of a dural sinus fistula is likely to show diffuse enlargement of the extraocular muscles resulting
from venous engorgement and an a characteristically enlarged superior ophthalmic vein. Orbital expansion with Valsalva is
characteristic of an orbital varix where there is dilation of preexisting venous channels. Phleboliths are also associated with
orbital varices. The internal carotid artery is minimally distensible and has a higher pressure than the dural sinus.
Question 99 of 130
When copious mucous refluxes from the superior canaliculus while irrigating through the inferior canaliculus, what is the
most likely site of obstruction?

Inferior canaliculus

Superior canaliculus

Nasolacrimal duct

Common internal punctum


Please select an answer
Feedback: Obstruction of either canaliculus would prevent any reflux with irrigation. Obstruction of the common internal
punctum would allow reflux of water and the minimal contents of the canaliculi, whereas nasolacrimal duct obstruction would
allow extravasation of the lacrimal sac content, which may consist of large amounts of mucus or pus.

Question 100 of 130


A 14-year-old boy has had transitory bilateral, painless (hours to days) eyelid edema which lasts over years. There is no
history of erythema, pruritus, or atopy. Examination shows baggy upper eyelid skin with a crepe paper-like appearance. For
this patient, what is the most likely diagnosis?

Contact dermatitis

Thyroid eye disease

Dermatochalasis

Blepharochalasis
Please select an answer
Feedback: Blepharochalasis is an idiopathic condition resulting in transient edema of the eyelids lasting several hours to
days. It may present in infancy, but commonly occurs in teen years and persists throughout life. Most cases occur
sporadically although an autosomal dominant pattern has been reported. Patients have recurrent episodes of edema not
associated with allergies. Recurrent stretching of the eyelid skin produces excess skin that can have a crepe paper-like
appearance. A contact allergy may also reccur and produce eyelid edema and stretching. However, erythema and pruritus
would be expected with an allergic reaction. Dermatochalasis refers to redundant skin resulting from involutional changes.
Usually there is no history of episodic edema leading to the occurrence of dermatochalasis. Both hyperthyroid and
hypothyroid patients can develop eyelid edema. Hyperthyroid patients may have acute inflammation of the orbital tissues,
resulting in secondary edema of the eyelids. Also, pseudoedema may occur secondary to weakening of the orbital septum
and prolapse of orbital fat forward into the eyelids. Hypothyroid patients may have myxedematous changes, resulting in
eyelid edema. Thyroid patients can have acute onset of edema, but it tends to last for weeks to months as opposed to the
hours or days noted in cases of blepharochalasis. Thyroid function tests are also helpful in distinguishing between patients
with Graves' disease and those with blepharochalasis.
Question 101 of 130
What is the preferred treatment for cicatricial ectropion?

Lateral tarsal strip plus repair of lower eyelid retractors

Lateral tarsal strip plus skin graft

Fascia lata suspension of the lower eyelid

Lateral tarsal strip plus medial spindle procedure


Please select an answer
Feedback: Cicatricial ectropion is due to a lack of vertical tissue length, usually of the anterior lamella. Placing a tarsal strip
with skin graft addresses the vertical shortage. A medial spindle procedure is useful for punctal ectropion. Lengthening of
the lower eyelid retractors can address entropion.. Fascia lata could be used to suspend the lower eyelid, but would not
result in lasting correction as it does not address the lack of vertical tissue shortage.

Question 102 of 130


A 35-year-old woman has suffered visual loss in the right eye over 3 years. The visual acuity in the left eye is 20/20.
Examination of the right eye shows visual acuity of 20/70, a right afferent pupillary defect, 3 mm axial proptosis and
bilaterally normal optic discs. appears normal. What is the most likelylikely diagnosis?

Adenoid cystic carcinoma of lacrimal gland

Orbital lymphoma

Optic nerve glioma

Optic nerve sheath meningioma


Please select an answer
Feedback: Optic nerve sheath meningiomas present with gradual painless loss of vision. Proptosis may be present. The
optic nerve head may appear normal, swollen, or atrophic. Adenoid cystic carcinoma of the lacrimal gland is associated with
rapid growth (history generally of less than 1 year) and early onset of pain. Orbital lymphomas mold to surrounding tissue
and therefore usually do not affect vision until proptosis is severe. Optic nerve gliomas occur predominately in children in the
first decade of life. Initial signs and symptoms include massive swelling and hemorrhage of the optic nerve head and severe
retro-orbital pain.

Question 103 of 130


For a basal-cell carcinoma of the eyelids, in what location is associated with the worst prognosis for recurrence and
mortality?

Lower eyelid margin

Lower eyelid (not involving lid margin)

Central upper eyelid

Medial canthus
Please select an answer
Feedback: Basal-cell carcinomas that arise in the medial canthal area and vertical midface (H zone) are more likely to be
deeply infiltrative than those from the eyelid margins. The lower eyelid, lower eyelid margin, and central upper eyelid would
not be in the highest risk group unless they extended into the medial canthus

Question 104 of 130


Which of the following tests for myasthenia gravis can precipitate respiratory arrest?

Tensilon test

Acetylcholine receptor antibody titer

Rest recovery

Ice test
Please select an answer
Feedback: Tensilon testing may be associated with cramping, bradycardia, or respiratory arrest, so the physician must be
prepared to administer atropine if needed. The other choices are all valid tests that are not associated with significant
adverse events and therefore may obviate the risk of tensilon testing.

Question 105 of 130


A patient with congenital ptosis has bilateral measurements of margin reflex distance +1 mm, lid fissures of 5 mm, and lid
excursions of 4 mm. What is the most appropriate surgical approach to treat the ptosis?

Bilateral Mullerectomy

Bilateral frontalis suspension

Bilateral maximal external levator resection

Bilateral Fasanella-Servat
Please select an answer
Feedback: The key measurement to planning repair of congenital ptosis in this patient is the eyelid excursion of 4 mm,
which is limited. Therefore this patient would likely require a sling procedure. This plan is supported by the marked ptosis,
which the other 2measurements suggest. The alternative procedures tighten the levator (levator resection), Muller's muscle
(mullerectomy), or both (Fasanell-Servat) and are inadequate for marked ptosis with poor levator function.
Question 106 of 130
A patient has an NLP and painful eye shortly after treatment for chronic endophthalmitis. When evaluating the surgical
options, which technique would minimize potential intracranial and orbital contamination?

Enucleation

Evisceration

Subtotal exenteration

Total exenteration
Please select an answer
Feedback:

Evisceration is preferred by some surgeons for endophthalmitis because the risk of orbital cellulitis and intracranial
extension is theoretically reduced.

Enucleation could potentially allow posterior spread of infection. Subtotal or total exenteration would involve removal of
healthy tissue and are therefore necessary.

Question 107 of 130


What tissue provides such a bright signal on a T1-weighted, magnetic resonance image (MRI) that it can obscure important
structures?

Bone

Fat

Vitreous

Lens
Please select an answer
Feedback:

Fat demonstrates a bright signal on T1-weighted magnetic resonance imaging. A special MRI sequence has been
developed to suppress the normal bright signal of fat on T1 images (fat suppression). None of the other tissues
interferes with the interpretation of other structures. Bone, lens and vitreous are seen well on T-1weighted images
but do not provide a bright signal and therefore do not obscure other tissues.
Question 108 of 130
What term describes an abnormally wide distance between the medial canthi in the presence of a normal interpupillary
distance?

Exorbitism

Hypertelorism

Telorbitism

Telecanthus
Please select an answer
Feedback:

Telecanthus refers to a wide intercanthal distance in the presence of a normal interpupillary distance (the medial
0
walls are normally positioned). Exorbitism refers to an angle between the lateral orbital walls that is greater than 90 ,
which is usually associated with shallow orbital depth. Telorbitism (hypertelorism) refers to a wider-than-normal
separation of the medial orbital walls. Exorbitism and hypertelorism generally refer to congenital abnormalities.

Question 109 of 130


What is the most common type of melanoma which occurs on the face?

Superficial spreading

Nodular

Acrolentiginous

Lentigo maligna
Please select an answer
Feedback: Lentigo maligna melanoma accounts for 90% of head and neck melanomas. Nodular melanoma accounts for
approximately 10% of cutaneous melanomas. Superficial spreading melanoma and acrolentiginous melanoma do not
commonly occur on the eyelid.
Question 110 of 130
A 1-week-old infant is having difficulty breathing due to bilateral congenital dacryocystocele. What management is needed?

Urgent decompression in the operating room

Systemic antibiotics

Topical antibiotics and massage

Bedside probing of the nasolacrimal duct

Please select an answer


Feedback: In this setting of bilateral intranasal cysts with breathing compromise or infection, urgent intervention in indicated.
Further, the potential need to secure the airway mandates this be performed in a controlled environment such as the
operating room. Following decompression of the cysts and control of bleeding, management consists of topical antibiotics
and massage for 2 weeks. If the cyst persists beyond 2 weeks of age, bedside probing of the nasolacrimal duct is
recommended.

Question 111 of 130


Patients with floppy eyelid syndrome may suffer from blepharoptosis in one or both eyes. Which of the following statements
accurately describes the surgical approach to their form of ptosis?

Frontalis suspension is often required to provide adequate eyelid elevation and contour.

Levator advancement/resection is not useful when treating this disorder.

Horizontal shortening of the upper lid is often enough to elevate the affected lid.

Lash ptosis does not respond to horizontal tightening of the eyelid.


Please select an answer
Feedback: Horizontal tightening of the floppy upper eyelid will frequently address both the ptosis and lash ptosis, although
levator advancement is sometimes necessary. Frontalis suspension is generally reserved for ptosis associated with floor
levator function, which is not characteristic of floppy eyelid syndrome.

Question 112 of 130


A mildly obese patient complains of chronic irritation in both eyes which is worse in the morning. In one eye, the patient has
ptosis. What question would address a risk factor for the patient's ptosis?

Do you sleep face down?

Do you suffer from recurring, unilateral facial spasms?

Do you suffer from dementia?

Do you have a history of Bell's palsy?


Please select an answer
Feedback: Floppy eyelid syndrome is a common cause of ptosis. Sleeping face down commonly exacerbates the symptoms
and signs. Facial spasms and dementia are not associated with unilateral ptosis and ocular irritation. Bell's palsy causes
eyelid retraction, not ptosis.

Question 113 of 130


A patient presents with the obesity, a soft rubbery tarsus, watery tearing, and mucus discharge from one eye. He prefers to
sleep in a face down position. What surgical treatment would be preferred to improve his symptom?

Horizontal eyelid tightening

Canthotomy of lateral canthal tendon

Canthotomy of superior limb of lateral canthus

Vertical shortening of tarsus

Please select an answer


Feedback: Floppy eyelid syndrome is characterized by chronic papillary conjunctivitis; easily everted, flacid upper eyelids;
and ocular irritation. Eyelash ptosis is common, and a marked decrease of elastic fibers in the tarsaus has been reported.
Ptosis rather than eyelid retraction is characteristic. The ptosis often can responds to horizontal lid tightening. Vertical
shortening of the tarsus is not typically needed. Canthotomies would aggrevate the lid laxity which characterizes this
condition. Canaliculitis has not been described, but mucus discharge is common.

Question 114 of 130


Which of the following signs is found in blepharochalasis syndrome?

Cicatricial entropion

Blepharoptosis

Hypertrophy of orbital fat pads

Thickened eyelid skin


Please select an answer
Feedback: Blepharochalasis is a rare, familial variant of angioneurotic edema. It can be associated with thin and wrinkled
skin, true ptosis, and atrophy of the orbital fat pads. Blepharochalasis is not associated with entropion of the eyelids, but
prolapse of the orbital portion of the lacrimal gland may occcur due to repeated attacks of edema.
Question 115 of 130
What is an advantage of evisceration over enucleation?

Lower risk of sympathetic ophthalmia

Better treatment of endophthalmitis

No need for placement of orbital implant

Better histopathologic examination of intraocular contents


Please select an answer
Feedback: Evisceration is preferred by some surgeons in the setting of endophthalmitis because there is theoretically a
reduced chance of orbital contamination with subsequent orbital cellulitis or intracranial extension of the infection.
Sympathetic ophthalmia has been reported with evisceration, presumably due to to a reaction to residual uveal tissue.
Evisceration should be avoided in suspected cases of malignancy because of inability to perform good
histologic examination of intraocular contents, sclera, and associated vessels. An implant is required in either case to
provide orbital volume.

Question 116 of 130


Which nutritional supplement should be avoided prior to eyelid and orbital surgery because of its effect on platelets?

Eicosapentaenoic acid (fish oil)

Echinacea

Glucosamine

Valerian
Please select an answer
Feedback: Fish oil inhibits platelet aggregation. Glucosamine may mimic human insulin and hypoglycemia. Echinacea
inhibits cytochrome P-450 and may adversely affect wound healing. Valerian inhibits the cytochrome P-450 enzyme system.

Question 117 of 130


A young patient presents with a high forehead and brow ptosis. Which of the following is the best surgical approach to treat
the brow ptosis?

Pretrichial endoscopic forehead lift

Coronal forehead lift

Direct eyebrow elevation

Midforehead lift
Please select an answer
Feedback: A pretrichial lift is the best cosmetic and functional choice in this situation. A coronal forehead lift is not optimal in
a patient who already has a high forehead because it would raise it even higher. Midforehead or direct eyebrow lifts are
suboptimal in young patients because the scars would be more evident without natural rhytids to disguise them.
Question 118 of 130
Limited downgaze is noted in a patient with a known floor fracture. What is the most likely cause of the downgaze deficit?

Disinsertion of the inferior rectus muscle from the globe

Orbital edema or hemorrhage affecting the inferior rectus muscle

Entrapment of the inferior oblique muscle in the floor fracture

Concomitant orbital roof fracture with entrapment of the superior rectus muscle
Please select an answer
Feedback: Restricted downgaze in the setting of an orbital floor fracture can be the result of entrapment of the inferior
rectus or edema and hemorrhage of the inferior rectus. Improvement usually occurs over 2 weeks if edema or hemorrhage
are responsible. Involvement of the superior rectus or inferior oblique is very rare in this setting, as is disinsertion of the
inferior rectus without direct penetrating trauma to the muscle.

Question 119 of 130


What pathologic finding is found in idiopathic orbital inflammation?

Monoclonal hypercellular lymphoid proliferation

Polyclonal hypercellular lymphoid proliferation

Granulomatous cellular infiltrate

Pleomorphic cellular infiltrate


Please select an answer
Feedback: Idiopathic orbital inflammation is characterized by a pleomorphic cellular infiltrate consisting of lymphocytes,
plasma cells, and eosinophils with variable degrees of reactive fibrosis. Monoclonal lymphoid infiltrates are more likely to
behave in a malignant fashion, and polyclonal lymphoid infiltrates are treated as a distinct entity with some potential for
malignant transformation. Granulomatous infiltrates are characteristic of other entities such as orbital sarcoidosis.

Question 120 of 130


In a young child with a subperiosteal orbital abcess, in what location(s) would medical therapy be preferred to surgical
drainage?

Lateral orbit

Orbital apex

Medial or inferior orbit

Superior orbit
Please select an answer
Feedback: Isolated inferior or medial subperiosteal orbital abscesses in children younger than age 9 with underlying
isolated ethmoid sinusitis, intact vision, and moderate proptosis typically respond to medical therapy. According to Garcia
and Harris guidelines, expectant therapy should not be offered in the presence of superior or lateral orbital involvement,
acute optic nerve or retinal compromise, or frontal sinusitis.
Question 121 of 130
What is the preferred treatment for lentigo maligna?

Serial biopsies

Observation

Cryotherapy

Excision with adequate surgical margins


Please select an answer

Question 122 of 130


What is the preferred management of pediatric optic nerve gliomas confined to orbit?

External radiation if confined to the optic nerve

Actinomycin D and vincristine

Surgical removal if vision unaffected

Observation
Please select an answer
Feedback:

Optic nerve gliomas that are confined to the orbit can be observed, particularly if vision is good. Radiation is used if the
tumor is unresectable because of chiasm or tract involvement and symptoms are progressing. Chemotherapy is effective in
progressive chiasmal or hypothalamic gliomas and may avoid complications of radiation. Surgery may be used for
progressive lesions expanding toward the chiasm.

Question 123 of 130


What is the prognosis of mucosal-associated lymphoid tissue (MALT) mass in the orbit?

Orbital enlargement, metastasis in half of patients within 10 years

Usually progresses to large-cell lymphoma

Rapid orbital enlargement, metastasis in nearly all patients within 10 years

Benign ocular and systemic morbidity


Please select an answer
Feedback: Although MALT lymphomas have a low grade of malignancy, long-term follow-up has demonstrated that at least
50% of patients develop a systemic disease at 10 years. MALT lymphomas may undergo spontaneous remission in 5-15%
of cases. They may undergo histologic transformation to a higher-grade lesion, usually of a large-cell type, in 15-20% of
cases. MALT lymphomas are not uniformly benign or uniformly aggressive
Question 124 of 130

A patient is asked to look from extreme downgaze to extreme upgaze. What term would you use to record the amount of
eyelid movement?

Levator function

Lid lag

Lagophthalmos

Mueller's muscle function


Please select an answer
Feedback: Measurement of the upper eyelid excursion is done by first fixating the frontalis muscle above the eyebrow and
the patient looking from extreme downgaze to extreme upgaze. The amount of movement is referred to as levator function.
Lid lag is immobility or lagging of the upper eyelid on the downward gaze. Lagophthalmos is the inability to close the eyes
completely. The contribution of Muller's muscle to eyelid elevation is not recorded as a distinct measurement.

Question 125 of 130


What is the most common complication following repair of total eyelid defects (upper and lower)?

Need for dacryocystorhinostomy

Eyelid rigidity manifested by ptosis and lagophthalmos

Proptosis

Corneal ulceration
Please select an answer
Feedback: Repair of total eyelid defects may result in eyelid rigidity or ectropion. Proptosis and nasolacrimal duct
obstruction are not common sequellae. Corneal ulceration can occur, but usually when lid defects or retraction complicated
eyelid rigidity or ectropion.

Question 126 of 130


What is the most important measurement to use when deciding whether a frontalis sling is the preferred treatment for
ptosis?

Upper eyelid excursion

Eyelid crease horizontal length

Palpebral fissure

Contralateral eyelid retraction


Please select an answer
Feedback: The frontalis sling is reserved for ptosis with poor levator function, as evidenced by poor upper eyelid excursion.
Palpebral fissure is indirectly useful because a small fissure implies a significant ptosis. The horizontal crease length is not
useful. Contralateral eyelid retraction implies only that the patient is attempting to lift both eyelids.

Question 127 of 130


For what type of facial spasm is magnetic resonance imaging useful?

Hemifacial spasm

Benign essential blepharospasm

Acute facial nerve palsy followed by aberrant regeneration

Blepharospasm associated with dry eyes


Please select an answer
Feedback: The differential diagnosis of hemifacial spasm, includes an ectatic vessel compressing the facial nerve and
pontine glioma, which may be detected by MRI. Neuroimaging is rarely indicated for benign essential blepharospasm
because the diagnosis is unlikely to be associated with an underlying disorder and can be made clinically. Acute facial nerve
palsies such as Bell's palsy do not require imaging. Dry eyes can present with blepharospasm-like symptoms, but
neuroimaging is not indicated.

Question 128 of 130

A febrile 65-year-old diabetic has orbital cellulitis with severe edema, areas of gray skin discoloration and tissue necrosis.
The sinuses are clear. What would be the preferred treatment?

Amphotericin B

Surgical debridement, broad-spectrum antibiotics, and probably ICU support

System steroids

Hyperbaric oxygen
Please select an answer
Feedback: The necrotizing cellulitis with clear sinuses and nasopharynx is most consistent with necrotizing fasciitis. Group
A Streptococcus is a common cause, but the diagnosis depends on cultures and tissue sampling. Fungal diseases almost
always originate in the sinuses or nasopharynx, and may respond to amphotericin B. Hyperbaric oxygen has been proposed
as a supplement for phycomycosis of the orbit. Steroids should be reserved for cases where it is clear the disease is
covered by an appropriate antibiotic.
Question 129 of 130
In the repair of a total eyelid defect from trauma, what is the preferred use of the disembodied tissue?

Reimplantation even if tissue has been ischemic for several hours

Avoidance of reimplantation because of infection potential

Avoidance of reimplantation because of graft-versus-host disease

Reimplantation with chemotherapy


Please select an answer
Feedback: Eyelid tissue is highly vascular, and replantation of convulsed eyelids can be successful hours after trauma.
Infection is possible, but not a reason to avoid repair. Graft-versus-host disease has not been reported. Chemotherapy is not
indicated.

Question 130 of 130


What clinical association is characteristic of lentigo maligna?

It presents as a thickened and nodular pigmented mass.

It is characterized by rapid growth.

It may progress to lentigo maligna melanoma.

Premalignant changes are confined to the clinically involved area.


Please select an answer
Feedback: Lentigo maligna is a flat, irregularly shaped unevenly pigmented, slowly enlarging lesion that may progress to
nodular lentigo maligna melanoma. Premalignant changes often extend beyond the clinically involved area.

Collected from www.aao.org (self assessment)

By Dr. AlBaraa AlQassimi

You might also like